Basic Radiology

Chapter 12. Brain and Its Coverings

BRAIN AND ITS COVERINGS: INTRODUCTION

Technologic advances in radiology during the past 30 years have vastly improved our ability to diagnose neurologic diseases. Prior to the introduction of computed tomography in 1974, neuroradiologic examinations of the brain consisted primarily of plain films of the skull, cerebral arteriography, pneumoencephalography, and conventional nuclear medicine studies. Unfortunately, these techniques, for the most part, provided only indirect information about suspected intracranial processes, were insensitive in detecting subtle or early brain lesions, or were potentially harmful to the patient. Computed tomography (CT) revolutionized the radiologic workup of central nervous system abnormalities because for the first time normal and abnormal structures could be directly visualized with minimal risk to the patient.

In the late 1980s, it became apparent that magnetic resonance (MR) imaging would become the procedure of choice for evaluating many neurologic disorders, as well as for demonstrating vascular flow phenomena. Since then, many technologic advances have been associated with this modality. These include improvements in magnet and coil design, decrease in imaging time, and the development of new pulse sequences such as MR angiography (MRA), MR spectroscopy (MRS), diffusion-weighted (DW) and perfusion-weighted (PW) MR imaging, and functional MR imaging (fMRI).

Revolutionary breakthroughs in CT scanning technology during the 1990s facilitated the development of advanced CT applications, namely, dynamic contrast-enhanced CT angiography (CTA) and CT perfusion (CTP). These techniques, which allow high spatial resolution imaging of the cervical and intracranial vasculature, are currently being used in the evaluation of the acute stroke patient in many medical centers. Furthermore, recent technologic advances in CT imaging have markedly decreased scan times and have allowed evaluation of very tiny anatomic structures due to improvements in spatial resolution.

Recent advances in nuclear medicine functional imaging techniques, including single photon emission computed tomography (SPECT) and positron emission tomography (PET); improvements in conventional arteriographic methods; and expansion of catheter-based therapeutic procedures have provided the neuroradiologist today with an even greater variety of strategies for diagnosing and treating neurologic abnormalities.

The main purpose of this chapter is to acquaint the reader with the major radiologic techniques currently being used to evaluate the brain and its coverings. The strengths and weaknesses of these techniques are discussed. Imaging anatomy of the brain and its coverings is briefly reviewed. Basic guidelines pertaining to technique selection for evaluating common neurologic conditions are provided. Finally, examples of common brain abnormalities are presented. It is assumed that readers have a basic understanding of neuroanatomy and neuropathology.

Although this chapter may give some insight into neuroradiologic study interpretation, that is not its primary goal. Rather, readers should expect to become reasonably familiar with the various techniques employed to examine the brain and should gain some idea about the appropriate ordering of examinations in specific clinical situations.

TECHNIQUES

Radiologic modalities useful in evaluating the brain and its coverings can be divided into two major groups: anatomic modalities and functional modalities. Anatomic modalities, which provide information mostly of a structural nature, include plain films of the skull, computed tomography, magnetic resonance imaging, cerebral arteriography, and ultrasonography. On the other hand, SPECT and PET imaging, CTP, DW and PW MR imaging, fMRI, and MRS are primarily functional modalities, which give information about brain perfusion or metabolism. Some techniques provide both anatomic and functional information. For example, cerebral arteriography depicts blood vessels supplying the brain but also allows us to estimate brain circulation time. Ultrasonography of the carotid bifurcation is another modality that provides both anatomic and functional information. A routine sonogram of the carotid bifurcation gives anatomic data that, when combined with Doppler data, readily provides information about blood flow.

The following discussion of current neuroradiologic techniques emphasizes relative examination cost and patient risk, along with the advantages and disadvantages of each technique. The normal imaging appearance of the brain and its coverings is also illustrated.

Plain Radiographs

Plain radiographs of the skull are obtained by placing a patient's head between an x-ray source and a recording device (i.e., x-ray film). Bones of the skull can block a large number of x-rays and, therefore, cast a white "shadow" on the x-ray film (Fig. 12–1). On the other hand, soft tissues such as scalp or brain cast little, if any, shadow on the film. Because the skull has a spherical shape, bones will be superimposed on one another. As a result, multiple routine views of the skull, including frontal, lateral, and axial projections, may be needed to adequately assess the calvarium and to accurately localize a lesion (Fig. 12–1). Even then, these studies can occasionally be tricky to interpret because of the large number of superimposed structures. The resultant skull radiograph primarily gives information about the bones of the skull, but no direct information about the intracranial contents. Indirect information about intracranial abnormalities can sometimes be obtained from the skull plain radiograph, although this information can be quite subtle, even in the setting of advanced disease. Skull plain radiographs have been largely replaced today by more sensitive techniques such as CT or MR imaging. Even in the setting of suspected skull fracture, plain radiographs are rarely indicated, because CT scans also show the fracture, as well as any intracranial abnormality that might require treatment.

Fig. 12–1.

Frontal A and lateral B plain films of the normal skull. Anatomic landmarks include ethmoid sinuses (E), frontal sinus (F), orbital roof (O), superior surface of the petrous portion of the temporal bone (P), sphenoid ridge (S), coronal suture (C), dens (D), anterior clinoid process (AC), dorsum sella (DS), sella turcica (ST), lambdoid suture (L), inner table of calvarium (IT), outer table of calvarium (OT), and sphenoid sinus (SS).

Computed Tomography

CT scans consist of computer-generated cross-sectional images obtained from a rotating x-ray beam and detector system. Recent advances in scanning technology now permit simultaneous acquisition of multiple images during a single rotation of the x-ray tube (e.g., up to 16 as of 2004) during a breath-hold. The resultant images, unlike plain films, exquisitely depict and differentiate between soft tissues, thus allowing direct visualization of intracranial contents and abnormalities associated with neurologic diseases. The contrast or brightness ("window" or "level," respectively) of these images can be adjusted to highlight particular tissues.

Typically, a head CT consists of images adjusted to emphasize soft-tissue detail (soft-tissue windows) as well as images adjusted to visualize bony detail (bone windows) (Fig. 12–2). As stated earlier, cortical bone appears white (has a high attenuation value or Hounsfield unit), whereas air within the paranasal sinuses appears black (has a low attenuation value) (Fig. 12–2). Cerebral white matter has a slightly lower Hounsfield number than does cerebral gray matter and consequently appears slightly darker than gray matter on a head CT scan (Fig. 12–2A ). Intracranial pathologic conditions can be either dark (low attenuation) or bright (high attenuation), depending on the particular abnormality. For example, acute intracranial hemorrhage is typically very bright, whereas an acute cerebral infarction demonstrates low attenuation when compared to the surrounding normal brain because of the presence of edema.

Fig. 12–2.

Normal axial head CT images. Appropriate window selection allows visualization of both intracranial contents A and bony calvarium B. Note differences in attenuation between gray matter (right basal ganglia, large arrow), white matter (left internal capsule, small black arrows), cerebrospinal fluid (CSF; frontal horn of the left lateral ventricle,white arrows), and bone (skull, arrowheads).

The CT technologist can change the slice thickness and angulation, among other technical factors, to alter the way an image appears. Axial images are most commonly obtained, but coronal images can be obtained with hyperextension of the patient's neck. Because CT images are computer generated, data making up the axial images can be reformatted in the coronal, sagittal, or oblique planes or as a 3-D image, although some resolution may be lost.

CT examinations are often performed after intravenous administration of an iodinated contrast agent. These agents "light up" or enhance normal blood vessels and dural sinuses, as well as intracranial structures that lack a blood–brain barrier (BBB), such as the pituitary gland, choroid plexus, or pineal gland. Pathologic conditions that interrupt the BBB also demonstrate enhancement after contrast material administration. For this reason, lesions that may be invisible on a noncontrast study are often obvious on a contrast-enhanced scan.

Volumetric images of intravenously injected contrast as it passes through the arterial circulation, or CTA, can now be routinely performed. These high spatial resolution 3-D CTA images of the cervical and intracranial vasculature have been implemented into recently developed acute stroke protocol examinations. In particular, CTA accurately identifies the location and extent of large vessel occlusions, which may predict response to reperfusion therapies. Tailored software analysis of CTA data can produce maps of whole-brain perfusion. These permit a physiologic determination of the extent of collateral circulation and volume of brain tissue affected by an ischemic event, as well as a measure of the severity of the insult. This information can additionally be supplemented by a more detailed, quantitative evaluation of the cerebral microvascular hemodynamics (CTP) during the early phase of bolus passage. The potential utility of CTA/CT perfusion imaging in the acute stroke patient is based on the high specificity and accuracy of this functional assessment.

Another recent application of CTA is in the screening evaluation of blunt cerebrovascular injury, including closed head injuries, seat belt abrasion (or other soft tissue injury) of the anterior neck, basilar skull fracture extending through the carotid canal, and cervical vertebral body fracture. It is an accurate technique for detecting internal carotid artery (ICA) dissections and for assessing stenoses, although evaluation is difficult in areas of surrounding dense bone as a result of associated "streak artifact." However, this noninvasive, relatively short imaging procedure rivals conventional angiographic methods, because it requires no patient transfer, and can sensitively identify vascular injury in relation to other associated brain insults, cervical spine injury, or facial or basilar skull fractures.

High-resolution data acquisition during the venous phase following intravenous contrast administration (CT venography) can be used to identify dural sinuses and cerebral veins, evaluate for dural venous sinus thrombosis, and distinguish partial sinus obstruction from venous occlusion in the setting of adjacent brain masses. CT venography can also differentiate slow flow from thrombosis, which may occasionally be difficult with MR techniques.

The major advantages of CT scanning are that it is inexpensive, widely available, can be used in patients with MR-incompatible hardware, and allows a relatively quick assessment of intracranial contents in the setting of a neurological deficit. The images obtained are very sensitive to the presence of acute hemorrhage and calcification, and images revealing exquisite bony detail of the skull and skull base can be acquired. Because of the configuration of the scanner, patients are reasonably accessible for monitoring during the examination.

CT scanners do have a number of disadvantages, however. Patients are exposed to ionizing radiation and iodine-based contrast agents (although lower doses of contrast are needed with the newer multidetector scanners). Imaging artifacts can interfere with accurate interpretation. In particular, images of the brain stem and posterior fossa are often degraded by "streak artifacts" from dense bone (Fig. 12–3). Streak artifacts from metallic objects (e.g., fillings, braces, surgical clips) can also obscure abnormalities. Images can be severely degraded by patient motion. Fortunately, unlike MR scans, individual CT images degraded by motion can be rapidly reacquired.

Fig. 12–3.

Streak artifacts (arrows) commonly obscure portions of the brain stem, posterior fossa, and temporal lobes on routine head CT scans.

Magnetic Resonance Imaging

One of the most exciting developments in radiology during the past 30 years has been the translation of nuclear magnetic resonance phenomena, initially used for probing the physicochemical structure of molecules, to imaging. The product of this application, MR imaging, has profoundly affected the radiologic evaluation of most neurologic disorders. MR examinations, like CT scans, consist of computer-reconstructed cross-sectional images (Fig. 12–4). In MR imaging, however, unlike CT scans or plain radiographs, the information collected is not x-ray beam attenuation. The MR image is a visual display of nuclear magnetic resonance data collected principally from nuclei within body tissues—especially hydrogen nuclei within water and fat molecules. Intrinsic tissue relaxation occurs by two major pathways, calledlongitudinal, or T1, and transverse, or T2, decay. MR imaging sequences that emphasize T1 decay are commonly referred to as T1-weighted, images; sequences that accentuate T2 relaxation properties are called T2-weighted images (Fig. 12–4). Most MR scans of the brain use both of these sequences, because certain abnormalities may only be obvious on one or the other. T2-weighted images are usually easy to identify because fluid (e.g., cerebrospinal, globe vitreous) is very bright; fluid on a T1-weighted scan is usually dark. Fat is bright on T1-weighted scans, but darker on T2-weighted images. On the other hand, both cortical bone and air are very dark on all imaging sequences. Brain tissue has intermediate intensity; vessels can have almost any signal, depending on the velocity of flowing blood.

Fig. 12–4.

Normal head MR images. Sagittal T1-weighted A, axial T1-weighted B, and axial T2-weighted C images. Note differences in signal between gray matter (large arrows), white matter (curved arrows), CSF (small arrowheads), fat (small arrows), and cortical bone (large arrowheads) on different pulse sequences. Normal structures include the genu (g) and splenium (s) of the corpus callosum (cc), fornix (f), optic chiasm (oc), pituitary gland (pit), midbrain (mb), pons (p), medulla (m), cerebellar vermis (Cb), straight sinus (SS), caudate head (c), putamen (pt), and thalamus (T).

The most commonly used, clinically approved contrast agent for MR imaging is gadopentetate-dimeglumine or Gd-DTPA, which is very well tolerated and extremely safe. Its major use in the central nervous system (CNS) is to improve lesion detectability by "lighting up" pathologic conditions that either lack a BBB or have a disrupted BBB.

Conventional MR imaging depicts excellent soft-tissue contrast. Traditionally, long image acquisition times, image artifacts related to patient motion, and the increased cost of scanning due to limited patient throughput have hampered the clinical utility of MR imaging. During the past 15 years, technical advances in gradient technology, coil design, image reconstruction algorithms, contrast administration protocols, and data acquisition strategies have accelerated the development and implementation of fast imaging methods. These techniques, including fast gradient echo imaging, fast spin echo imaging, FLAIR (fluid-attenuated inversion recovery), and echo planar imaging, have enabled substantial reductions in imaging time. Images may be acquired during a single breath-hold on a clinical scanner, eliminating respiratory and motion artifacts. Vessel conspicuity can be enhanced by application of fat-suppression sequences, which eliminate unwanted signal from background tissues. These improvements have led to a vast range of applications that were previously impractical, including high-resolution MRA, DW and PW MR imaging, MRS, fMRI, and real-time monitoring of interventional procedures.

Since its first clinical application nearly 15 years ago, MRA has proven to be a useful tool for evaluation of the cervical or intracranial carotid vasculature. MRA represents a class of techniques that utilize the MR scanner to noninvasively generate three-dimensional images of the carotid or vertebral-basilar circulations (Fig. 12–5). While a detailed discussion of these techniques is beyond the scope of this chapter, several comments are noteworthy. These methods permit distinction between blood flow and adjacent soft tissue, with or without administration of intravenous contrast. As noted above, revolutionary developments have permitted MRA images to be rapidly acquired with ever-improving temporal and spatial resolution.

Fig. 12–5.

Normal frontal view of intracranial MR angiogram at the level of the circle of Willis. Note the internal carotid artery (large arrow), anterior cerebral artery (small arrows), and middle cerebral artery (arrowheads).

Presently, MRA serves as one of the first-line studies for evaluation of arterial occlusive disease and for screening of intracranial aneurysms. These methods have largely replaced conventional arteriographic studies for evaluation of atherosclerotic disease, except in cases of critical stenosis (>70%). In these instances, the degree of luminal narrowing may be overestimated by MRA, and may require verification with a catheter-based study or Doppler ultrasound. Moreover, aneurysms detected on an intracranial MRA typically require a catheter-based study for detailing aneurysm size and orientation, for establishing the location of adjacent vessels and collateral flow, for confirming suspicious vascular dilatation, and for detecting the presence of vasospasm or additional aneurysms that may not be readily apparent on the MRA study. In an increasing number of cases, catheter-based studies will additionally be performed for coil embolization (obliteration) of detected aneurysms, rather than surgical clipping.

Molecular diffusion, the random translational movement of water and other small molecules in tissue, is thermally driven, and referred to as Brownian motion. Over a given time period, these random motions, expressed as molecular displacements, can be detected using specifically designed diffusion-sensitive MR sequences. A common application of diffusion imaging is the detection of early ischemic infarction, where the infarcted tissue "lights up" due to the "restricted diffusion" state within the intracellular compartment. Other applications of diffusion-sensitive sequences include differentiating cysts from solid tumors, as well as evaluating inflammatory/infectious conditions (encephalitis, abscess) or white matter abnormalities (hypertensive encephalopathy).

Perfusion MR imaging measures cerebral blood flow (CBF) at the capillary level of an organ or tissue region. Perfusion-weighted MR imaging has applications in the evaluation of a number of disease states, including cerebral ischemia and reperfusion, brain tumors (Fig. 12–6), epilepsy, and blood flow deficits in Alzheimer's disease. In addition, the close spatial coupling between brain activity and CBF permits the application of perfusion MR techniques to imaging brain function.

Fig. 12–6.

Perfusion (cerebral blood volume) mapping of a high-grade glioma in a patient who had previously undergone radiotherapy. A Axial postcontrast T1 image shows an enhancing lesion (arrow) present within the right posterior frontal deep white matter. B Axial cerebral blood volume map shows a corresponding high cerebral blood volume area measured (arrow). C PET image using radiolabeled glucose. High glucose uptake is demonstrated at the site of the lesion (arrow). (Courtesy of Dr. Hannu Aronen, Helsinki, Finland.)

Functional MR imaging is an important brain mapping technique that uses fast imaging techniques to depict regional cortical blood flow changes in space and time during performance of a particular task (e.g., flexion of the index finger). The utilization of this technique to localize brain activity is historically based on measurable increases in cerebral blood flow (and blood volume) with increased neural activity, referred to as neurovascular coupling. The hemodynamic response to a stimulus is not instantaneous, but on the order of a few seconds. Consequently, fMRI techniques are considered an indirect approach to imaging brain function, but provide excellent spatial resolution and can be precisely matched with anatomic structures. Changes in blood oxygenation and perfusion can be imaged using the fMRI technique, which has become the most widely used modality for depicting regional brain activation in response to sensorimotor or cognitive tasks.

An important clinical application of fMRI is presurgical mapping, whereby eloquent brain cortex can be defined in relation to mass lesions (Fig. 12–7). This allows for the judicious selection of an appropriate management strategy (surgical versus nonsurgical) according to the functional nature of the adjacent brain tissue. A second application involves determination of the cerebral hemisphere responsible for language and memory tasks in a patient with complex partial seizures, prior to undergoing temporal lobectomy. Additionally, several groups have reported successful functional activation studies for lateralizing language preoperatively utilizing fMRI.

Fig. 12–7.

Volume-rendered 3-D fMRI for presurgical brain tumor mapping. Note that the area of motor cortex activation partially overlaps the tumor volume within the left cerebral hemisphere. (Courtesy of Dr. Joseph Maldjian, Winston-Salem, NC.)

MR spectroscopy provides qualitative and quantitative information about brain metabolism and tissue composition. This functional analysis is based on detecting variations in the precession frequencies of spinning protons in a magnetic field. One factor influencing the precession or resonance frequency is the chemical environment of the individual proton. Protons in different cerebral metabolites can be sensitively discriminated on this basis, and the position of these metabolites can be displayed as a spectrum. The x-axis position of a given metabolite reflects the degree of "chemical shift" of the metabolite with respect to a designated reference metabolite, and it is expressed in units of parts per million (ppm). The area under the peak is determined by the number of protons that contribute to the MR signal.

The major metabolites detected in the CNS are N-acetyl aspartate (NAA), a neuronal marker; choline, a marker for cellularity and cell membrane turnover; creatine, a marker for energy metabolism; and lactate, a marker for anaerobic metabolism. In addition to these metabolites, others have been assessed, including alanine, glutamine, myoinositol, and succinate, using various MR strategies. Presently, MRS is being used in clinical practice to provide functional information regarding many CNS abnormalities, and it complements the conventional MR imaging study. A common application relates to the pre- and post-treatment evaluation of brain tumors, with MRS playing an important role in assessing for residual or recurrent tumor following surgical resection.

MR imaging offers a number of advantages over CT in the workup of patients with neurologic disease. Its soft-tissue contrast resolution is superior to that of CT, and lesions that may be subtle or invisible on CT are frequently obvious on MR imaging. MR imaging also allows acquisition of multiplanar views in the sagittal, axial, coronal, and oblique projections that may be impossible to obtain with CT. Furthermore, MR imaging gives information about blood flow without the need for a contrast agent, and bony streak artifacts that obscure lesions of the brain stem and cerebellum on CT scans are not present on MR images. Finally, MR imaging does not expose the patient to ionizing radiation.

Cerebral Arteriography

Cerebral arteriography involves the injection of water-soluble contrast material into a carotid or vertebral artery. Contrast material is injected into the desired vessel via a small catheter, which has been introduced into the body through the femoral or brachial artery. Information about the arterial, capillary, or venous circulation of the brain is recorded on serial plain films or digitized for viewing on a TV monitor or for storage within a computer (Fig. 12–8).

Fig. 12–8.

Normal cerebral arteriogram. A Lateral view of the cervical carotid artery. Catheter is located within the common carotid artery, and contrast material fills internal (arrows) and external (arrowheads) carotid arteries. B Lateral view of the head after injection of the carotid artery (arrow). Note anterior cerebral (A), ophthalmic (O), posterior communicating (PC), and middle cerebral (M) branches.

Cerebral arteriograms are expensive (two to three times as much as MR examinations) and are relatively more risky procedures than other neuroradiologic studies. The major risk of the procedure is stroke, which may occur in 1 of every 1000 patients. Stroke during cerebral arteriography occurs either from an embolic event (e.g., inadvertent injection of air, thrombus formation on catheter tip, atherosclerotic plaque dislodged by catheter manipulation) or from catheter-related local vessel trauma (e.g., dissections, occlusions).

Although CA is an invasive study with well-known risks, it is invaluable in the workup of vascular diseases affecting the CNS. It is the gold standard for assessing vascular stenosis and atherosclerosis or vasculitis, and it is indispensable in identifying and evaluating cerebral aneurysms and certain intracranial vascular malformations or fistulae. It is useful in assessing carotid or vertebral artery integrity after trauma to the neck, especially in the setting of acute neurologic deficit. Finally, it is unsurpassed for showing vascular anatomy of the brain and is, therefore, useful as a preoperative road map. CT and MR imaging have replaced cerebral arteriography in the workup of most other neurologic diseases and, as previously mentioned, rival cerebral arteriography in the detection of arterial occlusive disease, aneurysms, and vascular injury following blunt trauma to the neck.

The field of interventional neuroradiology continues to grow and exert considerable impact on the diagnosis and treatment of certain CNS diseases. New catheter designs and materials, recently developed endovascular devices (extracranial/intracranial stents), and an increasing number of trained specialists performing endovascular procedures have led to novel therapeutic applications and approaches for managing previously untreatable conditions. Endovascular diagnostic and therapeutic procedures, based on fundamental cerebral arteriography principles, have gained widespread acceptance and, in some cases, rival traditional neurosurgical approaches in terms of complication rates, clinical outcomes, and long-term survival benefit. Although a full discussion of these techniques is beyond the scope of this chapter, they include thrombolysis of intracranial clot in the setting of acute infarction or dural sinus thrombosis, embolization (obliteration) of intracranial aneurysms using thrombosing material (i.e., coils), carotid artery angioplasty and/or stent placement for critical stenotic narrowing or radiation-induced arterial stricture, preoperative or definitive devascularization of a hypervascular mass or arteriovenous malformation, embolization of small, bleeding external carotid artery branches in epistaxis, balloon occlusion tests of the carotid artery, and endovascular treatment of vasospasm. Embolization materials include particulate emboli, liquid adhesive glues, and various coils.

Ultrasonography

Ultrasonography is the diagnostic application of ultrasound to the human body. Major applications of ultrasonography in CNS disease include gray-scale imaging and Doppler evaluation of carotid artery patency and flow in the setting of atherosclerosis, assesssment of vasospasm in the setting of subarachnoid hemorrhage using transcranial Doppler, screening evaluation of intracranial abnormalities in the newborn and young infant (Fig. 12–9), and detection of intracranial hemorrhage in premature infants prior to extracorporeal membrane oxygenation therapy. Ultrasonography has also been used intraoperatively to demonstrate the spinal cord and surrounding structures during spine surgery and to define tumor and cyst margins during craniotomies.

Fig. 12–9.

Coronal A and sagittal B head ultrasound images of a neonate. Normal structures include the corpus callosum (CC), lateral ventricle (LV), cavum septum pellucidum (CS), sylvian fissure (SF), third ventricle (3V), fourth ventricle (4V), temporal lobe (T), frontal lobe (FR), occipital lobe (OCC), cerebellum (CER), and thalamus (TH).

Transcranial Doppler is a recently developed tool in the evaluation of cerebrovascular disorders. It uses low-frequency sound waves to adequately penetrate the skull, and produces spectral waveforms of the major intracranial vessels for evaluation of flow velocity, direction, amplitude, and pulsatility. Present clinical applications include diagnosis of cerebral vasospasm, evaluation of stroke and transient ischemic attack, detection of intracranial emboli, serial monitoring of vasculitis in children with sickle cell disease, and assessment of intracranial pressure and cerebral blood flow changes in patients with head injury or mass lesions.

Ultrasound examinations, although moderately expensive, are virtually risk free to the patient, involve no ionizing radiation, and are portable (i.e., can be performed at the bedside). However, examination quality and therefore diagnostic accuracy are operator dependent. Also, the heavy reliance of ultrasonography on the presence of an adequate "acoustic window" through which an examination can be performed diminishes its usefulness in examining the brain after the fontanelles close in infancy. Finally, to the untrained eye, anatomic structures and pathologic processes as depicted by US are not as readily apparent as they are on CT or MR images.

Single Photon Emission Computed Tomography

SPECT uses a rotating gamma camera to reconstruct cross-sectional images of the distribution of a radioactive pharmaceutical that has been administered to a patient (usually intravenously). For brain imaging, radioactive iodine (123I) or technetium (99mTc) is combined with a compound that rapidly crosses the BBB and localizes within brain tissue in proportion to regional blood flow. The rotating gamma camera detects gamma rays emitted by the radiopharmaceutical and produces cross-sectional images of the brain that are really a map of brain perfusion (Fig. 12–10). SPECT imaging also gives indirect information about brain metabolism, because perfusion is usually highest to parts of the brain with high metabolic activity and lowest to areas with low metabolic demand. Normal SPECT examinations demonstrate activity concentrated primarily in areas of high perfusion/metabolism, such as the cortical and deep gray matter (Fig. 12–10).

Fig. 12–10.

Axial SPECT image of normal cerebral perfusion. Note that perfusion is greatest to gray matter structures, including the cerebral cortex (large arrows) and deep gray nuclei (arrowheads). White matter and ventricles are nearly invisible because of low or no perfusion. The greatest perfusion is to the visual cortex area (small arrows).

SPECT studies are moderately expensive (as much as or more than brain MR imaging), and, as expected, they provide limited anatomic information. SPECT also exposes patients to ionizing radiation. Because patients rarely have allergic reactions to the radiopharmaceuticals used, the examination is of low risk. While SPECT provides critical information regarding regional cerebral perfusion, particularly in the setting of stroke, this information can be more readily obtained during CTA/CT perfusion or MR perfusion acquisitions. SPECT has also been used with varying degrees of success in the workup of patients with epilepsy or dementia.

Positron Emission Tomography

PET scans consist of computer-generated cross-sectional images of the distribution and local concentration of a radiopharmaceutical. This technique is very similar to SPECT imaging. The main difference is that PET studies use radiopharmaceuticals labeled with a cyclotron-produced positron emitter. These agents are very expensive to produce and have a very short half-life (on the order of seconds to minutes). The most widely used radiotracer is 18F-deoxyglucose. PET scanning with this agent gives a measurement of brain glucose metabolism. Other agents are useful in assessing regional cerebral blood flow, neuroreceptor function, and the like.

At first glance, PET scans resemble CT scans. Images can be viewed on a TV monitor or on x-ray film. Areas of high metabolic activity (i.e., cerebral cortex, deep gray nuclei) demonstrate greater radiopharmaceutical uptake than do areas of low metabolic activity, such as white matter or cerebrospinal fluid (Fig. 12–11). The bones of the skull and scalp soft tissues are, for the most part, invisible.

Fig. 12–11.

Normal axial image of brain PET scan. As in the SPECT study (Fig. 12–10), areas of high activity correspond to metabolically active gray matter (arrows), especially the visual cortex (arrowheads).

PET scans are very expensive, costing approximately twice as much as an MR scan. This expense is directly related to the high cost of operating the PET facility, which requires on-site physicists as well as an on-site cyclotron for radiotracer production. Therefore, PET scanning is not generally available in community hospitals. Although patients undergoing PET examinations are exposed to ionizing radiation, the overall risk to the patient is low. Anatomic resolution, although not as good as with CT or MR imaging, is better than with SPECT imaging. The major advantage of PET imaging is that it is extremely versatile, providing in vivo information about brain perfusion, glucose metabolism, receptor density, and, ultimately, brain function.

PET provides useful information in the setting of stroke, epilepsy, dementia, and tumors. At present, the two main indications are in the workup of patients with complex partial seizures and in identifying tumor recurrence in patients who have undergone surgery, radiation therapy, or both, for brain tumors.

TECHNIQUE SELECTION

The primary goal of a radiologic examination is to provide useful information for disease management. Radiologic studies can provide a diagnosis or can give information about disease extent or response to treatment. In the present medical climate, it has also become imperative that radiologic workups be performed efficiently and in a cost-effective manner. This requirement presents a problem for clinicians trying to decide which test to order in a given clinical situation.

The major strengths and weaknesses of neuroradiologic examinations were discussed earlier in this chapter. The following brief discussion concerns the appropriate ordering of examinations in clinical situations. Several points should be emphasized. First, although a recommended modality may clearly be superior to another in evaluating a particular neurologic condition, the choice of examination is not always obvious before the diagnosis is established. For example, in patients with nonfocal headache, MR scans are more sensitive than CT scans for detecting most intracranial abnormalities. However, if the headache is produced by subarachnoid hemorrhage, CT would be a much better examination than MR imaging, since subarachnoid hemorrhage is nearly invisible on MR images.

Choice of examinations may also be limited by what is locally available. If MR imaging is unavailable or if the MR scanner is of poor quality or if the interpreting radiologist is inadequately trained in MR image interpretation, then CT would be an excellent examination for evaluating most neurologic disorders.

Next, it is important to realize that the least expensive examination is not always the best first choice, even in this cost-conscious age. For example, most suspected skull fractures should be evaluated with CT scanning and not with plain films, despite the significant cost differential, because what is really important in management decisions is not the fracture itself but the potential underlying brain injury. Some neurologic diseases require multiple radiologic studies for accurate evaluation. Complex partial seizures refractory to medical management frequently require multiple examinations to localize the seizure focus prior to temporal lobectomy. Such a workup normally includes MR imaging and, ictal/interictal SPECT and/or PET scanning of the brain, as well as a cerebral arteriogram to identify cerebral dominance.

Finally, certain examinations are contraindicated in certain patients, and an alternative test must suffice. Patients with ferromagnetic cerebral aneurysm clips or pacemakers should not undergo MR imaging. Patients with a strong history of allergic reaction to iodinated contrast media should not routinely undergo contrast-enhanced CT scanning, unless they are pretreated with anti-inflammatory agents (i.e., steroids). MR scanning is frequently unsuccessful in claustrophobic or uncooperative patients unless they are sedated.

Congenital Anomalies

Congenital anomalies of the brain are best evaluated by MR imaging. MR imaging is the very best examination for demonstrating intracranial anatomy. It provides excellent discrimination between gray matter and white matter, superb views of the posterior fossa and craniocervical junction, and, most importantly, the ability to view the brain in any plane. MR imaging has, for all practical purposes, completely replaced CT for this indication. The one exception is in evaluation of skull abnormalities such as suspected fusion of the sutures.

Craniocerebral Trauma

CT is the preferred modality for studying practically all acute head injuries. Examination times are short, intracranial hemorrhage is well demonstrated, and skull fractures are readily apparent. Unstable patients can also be easily monitored. Intravenous administration of contrast agents is unnecessary in this setting. Occasionally cerebral arteriography is performed to look for carotid or vertebral artery injury when there has been penetrating trauma to the neck. Similarly, a CTA, MRA, or catheter-based study may be required to evaluate suspected carotid artery dissection associated with blunt head trauma or to assess carotid laceration in skull-base fractures.

While MR imaging is not routinely performed in the acute trauma setting, it may sometimes be helpful in patients with neurologic deficits unexplained by a head CT examination. For example, traumatic brain stem hemorrhages are often difficult to see on CT scans but are usually quite obvious on MR images. MR imaging is also useful in demonstrating tiny shear lesions within the brain in diffuse axonal injury and in assessing the brain in remote head trauma.

Intracranial Hemorrhage

The best examination to perform in most cases of suspected acute intracranial hemorrhage is a head CT scan. CT scans can be obtained quickly, allowing rapid initiation of treatment, and they are very good at demonstrating all types of intracranial hemorrhage, including subarachnoid blood. MR imaging takes much longer to perform in a potentially unstable patient, and subarachnoid hemorrhage may be difficult to see. MR imaging is more useful in the subacute or chronic setting, especially since it gives information about when a hemorrhagic event occurred. This information might be useful in such settings as nonaccidental head trauma (e.g., child abuse). MR imaging is also very sensitive to petechial hemorrhage that frequently accompanies a cerebral infarction and could potentially help to identify an underlying cause for an intracranial hemorrhage (e.g., tumor, arteriovenous malformation, occluded dural sinus). Finally, because most nontraumatic subarachnoid hemorrhage occurs secondary to a ruptured intracranial aneurysm, cerebral arteriography is routinely performed after detection of subarachnoid hemorrhage. More recently, obtaining a CTA study following the acquisition of a conventional head CT in the emergency department has promoted the concept of "one-stop shopping" for aneurysm detection and characterization prior to determining the appropriate management.

Aneurysms

The vast majority of intracranial aneurysms in which surgical intervention is planned require evaluation by cerebral arteriography. Cerebral arteriography not only allows aneurysm identification, but also provides other critical preoperative information such as aneurysm orientation, presence of vasospasm, location of adjacent vessels, and collateral intracranial circulation. Arteriography also helps to determine which aneurysm has bled when more than one aneurysm is present. As mentioned earlier, interventional neuroradiologists can treat aneurysms, usually in nonsurgical patients, by placing thrombosing material (i.e., coils) within the aneurysm itself via an endovascular approach.

Although most patients with symptomatic cerebral aneurysms present with subarachnoid hemorrhage, some aneurysms act like intracranial masses. These situations usually warrant evaluation by MR imaging as a first examination. The same is sometimes true with posterior communicating artery aneurysms (which can produce symptoms related to the adjacent third cranial nerve) or with aneurysms arising from the ICA as it courses through the cavernous sinus (which can affect any of the cranial nerves that lie within this structure, including cranial nerves III, IV, V, or VI).

Vascular Malformations

Patients with a vascular malformation (e.g., arteriovenous malformation, cavernous angioma, venous angioma, capillary telangiectasia) often seek medical attention after an intracranial hemorrhage or a seizure. In this setting, the first test that should be performed is either a CT examination (to look for intracranial hemorrhage) or MR imaging. Although an intracranial hemorrhage is usually very obvious on a CT scan, the vascular malformation itself may be difficult, if not impossible, to see unless intravenous contrast material is administered. MR imaging, on the other hand, is quite sensitive for detecting vascular malformations, whether they have bled or not. As can be seen, the choice of the initial examination for evaluation of a vascular malformation can be difficult. Usually, patients undergo noncontrast head CT scanning to look for intracranial hemorrhage when they come to the emergency department. Head CT is followed by gadolinium-enhanced MR imaging to further characterize the CT findings. If a high-flow true arteriovenous malformation is suspected, either clinically or from a cross-sectional imaging study, then cerebral arteriography is performed prior to initiation of treatment. MR angiography may someday replace conventional arteriography in the workup of these lesions, as with aneurysms.

Infarction

Most patients today with suspected cerebral infarction undergo CT scanning in the acute setting, even though infarctions are demonstrated earlier and are more obvious on MR imaging. So why is CT usually performed first? The answer is that clinicians who manage stroke patients are not so interested in seeing the infarct itself. Infarct location is usually suspected from the physical examination and acute infarcts may not even be visible on CT scans for 12 to 24 hours after onset of stroke symptoms. Clinicians are very interested, though, to know if a stroke is secondary to something besides an infarct (e.g., intracranial hemorrhage, brain tumor), or if an infarct is hemorrhagic, since thrombolytic agents would be contraindicated in this setting. CT can quickly answer both of these questions. MR imaging, specifically diffusion-weighted imaging, can sensitively detect acute infarctions and is typically ordered in cases of high clinical suspicion, when the initial CT study is nondiagnostic or when brain stem or posterior fossa infarcts are suspected.

The underlying cause of most cerebral infarctions is thromboembolism related to atherosclerosis. A CT/CTA or MR/MRA (including DW and PW MR imaging) study may provide a positive imaging diagnosis of brain infarction, reveal the extent and location of vessel occlusion, demonstrate the volume and severity of ischemic tissue, predict final infarct size and clinical prognosis, and, in the case of MR, potentially identify poorly perfused tissue at risk of infarction. Ultrasonography and cerebral arteriography can also be performed in the setting of stroke or transient ischemic attack to identify vascular stenoses or occlusions; these examinations are usually reserved for patients who might be candidates for carotid endarterectomy. Functional examinations (SPECT and PET) have also been used in patients with stroke-like symptoms to identify regions of the brain at risk for infarction. These studies are not widely available, and therefore do not enter into the imaging algorithm for most stroke patients.

Brain Tumors and Tumor-Like Conditions

The best examination to order in the setting of suspected brain tumor is a contrast-enhanced MR scan. This is true for primary neoplasms as well as for metastatic disease. MR imaging is especially useful in identifying tumors of the pituitary region, brain stem, and posterior fossa, including the cerebellopontine angle.

Although MR imaging is the preferred examination for intracranial neoplasms, it is occasionally supplemented by a CT scan, which can give important pre- treatment information not provided by MR images. For example, CT can demonstrate tumor calcification, occasionally a useful factor in differentiating between types of neoplasms. Also, CT is very useful in identifying bone destruction in skull base lesions.

In most medical centers, MR imaging and often MRS are performed to assess brain tumor response to treatment. PET scanning has also been used for this purpose. MRS and PET scanning can frequently differentiate recurrent tumor from postradiation tissue necrosis, which can mimic tumor on an MR or a CT scan.

Cerebral arteriography is rarely performed for brain tumor evaluation today except to map the blood supply of very vascular tumors (i.e., juvenile angiofibromas, paragangliomas) preoperatively. Such lesions can also be devascularized prior to surgery to minimize blood loss by injecting various materials into feeding vessels to occlude them.

Infection

Intracranial infections are best evaluated by contrast-enhanced MR imaging. Abscesses, cerebritis, subdural empyemas, and other infectious or inflammatory processes are all very well demonstrated. MR imaging is especially useful in assessing patients with the acquired immune deficiency syndrome (AIDS). Not only does it allow identification of secondary infections (e.g., toxoplasmosis, cryptococcosis, progressive multifocal leukoencephalopathy), but it is also exquisitely sensitive to the white matter changes produced by the human immunodeficiency virus (HIV) itself. CT scanning is less sensitive than MR imaging in the detection of intracranial infections and should be reserved for patients in whom MR imaging is contraindicated. Cerebral arteriography is only useful in one particular situation, suspected vasculitis. Involvement of brain arteries and arterioles in this condition requires arteriography for diagnostic confirmation.

Inherited and Acquired Metabolic, White Matter, and Neurodegenerative Diseases

As with suspected intracranial infections, this large and diverse group of diseases is best evaluated with MR imaging, which sensitively detects white matter abnormalities. In fact, one of the very first clear indications for MR imaging was in the workup of suspected multiple sclerosis. Although brain abnormalities in these conditions may be quite obvious on MR imaging, there is one problem: Many of these conditions appear very similar and an exact diagnosis may not be possible. In patients with dementia and suspected neurodegenerative disease, PET imaging is currently the procedure of choice for diagnostic evaluation.

EXERCISE 12-1: CONGENITAL ANOMALIES

Clinical Histories:

Case 12-1. A 2-day-old male infant presents with multiple craniofacial deformities, including frontal bossing, microcephaly, and a fleshy mass on the bridge of the nose. Sagittal and coronal T1-weighted MR images of the brain are shown in Fig. 12–12A,B .

Case 12-2. A 15-month-old female infant presents with new onset of seizures. Axial T1- and T2-weighted MR images are shown in Fig. 12–13A,B .

Fig. 12–12.

Fig. 12–13.

Questions:

12-1. In Case 12-1, what is the major abnormality?

A. Enlarged ventricles

B. Cyst in the posterior fossa

C. Lack of brain cleavage into two hemispheres

D. Herniation of intracranial contents through a skull defect

E. Abnormal migration of gray matter

12-2. In Case 12-2, what is the etiology of the patient's seizures?

A. Brain tumor

B. Gray matter in the wrong place (i.e., heterotopic gray matter)

C. Congenital infection

D. Nodules along ventricles in a patient with tuberous sclerosis

E. Infarction of periventricular white matter

Radiologic Findings:

12-1. In this case, the corpus callosum (curved arrow) is absent on the sagittal T1-weighted MR image (Fig. 12–12A). Also note other midline abnormalities, including abnormal tissue at the bridge of the nose (large arrow) and a posterior cyst (small arrows). The coronal T1-weighted MR image (Fig. 12–12B ) demonstrates a monoventricle (small arrows) and thalamic fusion (curved arrow). Also note the lack of separation of the two hemispheres (large arrow). (C is the correct answer to Question 12-1.)

12-2. In this case, T1-weighted (Fig. 12–13A ) and T2-weighted (Fig. 12–13B ) MR images show abnormal tissue lining the lateral ventricle (arrows). Signal of this tissue follows that of normal gray matter (arrowheads) on both T1- and T2-weighted images. (B is the correct answer to Question 12-2.)

Discussion:

Two common reasons for performing MR scans in young infants are illustrated by the cases in this section. Infants with craniofacial anomalies frequently have underlying congenital malformations of the CNS. Seizures, too, may be the first sign of an underlying brain malformation. As discussed in the section on technique selection, whenever a congenital brain anomaly is suspected, MR imaging is the best examination to perform.

Insults to the developing brain lead to predictable alterations of brain morphology. By analyzing patterns of altered brain morphology, we can often determine which stage of CNS development has been disrupted. This analysis, combined with a knowledge of neuroembryology, has allowed for the development of systems to classify congenital anomalies of the CNS. One simplified classification system divides congenital malformations into disorders of organogenesis (which include abnormalities of neural tube closure, diverticulation/cleavage, sulcation/cellular migration, and size, as well as destructive lesions acquired in utero), disorders of histogenesis (i.e., neurocutaneous syndromes), and disorders of cytogenesis (i.e., congenital neoplasms). Readers are referred to the Bibliography at the end of this chapter for further information on this topic.

The patient in Case 12-1 has alobar holoprosencephaly, a classic example of disordered ventral induction. In this condition, there is complete (alobar) or partial (semilobar, lobar) failure of separation of the forebrain (prosencephalon) into two hemispheres. In alobar holoprosencephaly, the most severe form of this disorder, there is no separation of the two hemispheres at all. The thalami are fused, a central monoventricle is present, and there is no corpus callosum. Infants with this form of holoprosencephaly frequently have severe facial anomalies.

In Case 12-2, the patient has heterotopic gray matter lining the lateral ventricles. This congenital anomaly is one type of disordered cellular migration. Neurons that make up the gray matter of the cerebral cortex actually develop along the edges of the lateral and third ventricles within the so-called germinal matrix zone. They then migrate outward to their final cortical location. If this normal neuronal migration is disrupted, a normal cortex may not develop, and clumps of gray matter may be present in abnormal locations along the migration route. Collections of these normal neurons in abnormal locations are called gray matter heterotopias.

Several different types of heterotopias have been described. The case presented in this section demonstrates a focal nodular gray matter heterotopia involving the subependymal region at the edge of the lateral ventricles. Seizures frequently occur in patients with this condition, as in the patient in Case 12-2. Because MR imaging usually provides an exact diagnosis of this condition, biopsies of CNS tissue are unnecessary.

In contrast to focal nodular heterotopias, diffuse (or laminar) heterotopias are commonly seen within or adjacent to the cortex, while "band"-type heterotopias are located deep to the normal cortex, in a subcortical location, separated by a thin interface of white matter (Fig. 12–14). Band-type heterotopias are well defined, with smooth margins, demonstrating signal intensities identical to normal gray matter. Mass effect on the underlying white matter or deep gray structures may be seen, and the sulcation pattern of the brain superficial to the heterotopia may be abnormal. Associated CNS anomalies may be present, such as agenesis of the corpus callosum, holoprosencephaly, or herniation of brain tissue (encephaloceles). While, at first glance, the cortex may appear to be markedly thickened, closer examination will reveal an additional band of gray matter in a subcortical location, that may or may not demonstrate increased 18F-FDG activity on a PET scan. This band of heterotopia is known to be associated with intractable seizures, occurring earlier than in the focal type, as well as severe developmental delay.

Fig. 12–14.

A Postcontrast coronal T1-weighted image of the brain in a 32-year-old patient with intractable seizures. An additional circumferential band of gray matter is seen (arrows) deep to the normal gray matter within the occipital region. This finding was noted to be diffusely present throughout the remaining brain parenchyma (not shown). B The corresponding PET image in the same patient reveals increased activity of the band heterotopia relative to the adjacent normal cortex (arrows), of unclear significance.

Several types of Chiari malformations were initially described by the German pathologist Hans Chiari, who classified these congenital hindbrain anomalies into three types. In each case, abnormal descent of cerebellar tissue into the cervical canal is demonstated. A Chiari I malformation consists of elongated peg-like cerebellar tonsils below the foramen magnum, while the brain stem is normal in location. In cases where the tonsils, vermis, pons, medulla, and an elongated fourth ventricle are displaced inferiorly into the upper cervical canal, a Chiari II malformation is diagnosed (Fig. 12–15). Chiari III malformations are associated with occipital or high cervical encephaloceles, containing cerebellar tissue, with or without brain stem.

Fig. 12–15.

Uninfused sagittal T1-weighted A and coronal FLAIR B imaging in a 30-year-old patient with a Chiari II malformation.A A small posterior fossa is present, resulting in cerebellar tonsillar ectopia (long arrow), towering of the cerebellum (short arrow), beaking of the tectum (curved arrow), and compression of the fourth ventricle (arrowhead) with resulting hydrocephalus. Partial agenesis of the rostrum and splenium of the corpus callosum is noted. B Cerebellar tonsillar ectopia into the foramen magnum is demonstrated (arrows).

In a Chiari II malformation, displacement of the fourth ventricle may elongate the aqueduct and compress the fourth ventricle, with resulting hydrocephalus. Because the whole posterior fossa is smaller than expected, the posterior fossa structures have a distorted appearance, and also assume abnormal locations. The superior cerebellum towers superiorly through a widened tentorium incisura, with the remainder of the cerebellum wrapping around the brain stem. The tectum of the midbrain is beaked, and the massa intermedia is enlarged. A kink may be present at the cervicomedullary junction. Complete agenesis of the corpus callosum may be present in up to one-third of cases, and partial agenesis may be seen in 75% to 90% of cases, predominantly involving the splenium.

Disorders of histogenesis include the neurocutaneous syndromes, which are a heterogeneous group of disorders with CNS and, for the most part, cutaneous manifestations. Visceral and connective tissue abnormalities may be prominent. Common disorders within this group include neurofibromatosis types I and II, tuberous sclerosis, von Hippel-Lindau disease, and Sturge-Weber syndrome, where the abnormal lesions corresponding to these entities are neurogenic tumors, tubers, hemangioblastomas, and angiomas, respectively.

Neurofibromatosis type 1 is the most common of all the neurocutaneous syndromes, accounting for 90% of all neurofibromatosis cases, and is the only such entity discussed here. It is transmitted on the long arm of chromosome 17 and is a disease of childhood. Autosomal dominant transmission occurs in 50%, while the remainder sporadically appear as new mutations in a patient with no known family history of the disease. The diagnosis is established when two or more of the following criteria are present: (1) six or more café-au-lait spots (brown skin pigmentation), (2) two or more Lisch nodules (hamartomas) of the iris, (3) two or more neurofibromas, (4) one or more plexiform neurofibromas, (5) axillary freckling, (6) one or more bone dysplasias (i.e., dysplasia of the greater sphenoid wing), (7) optic nerve glioma, or (8) first-degree relative with neurofibromatosis type 1.

The optic pathway gliomas are generally nonaggressive (low-grade) pilocytic astrocytomas that present in childhood and may not affect vision until greatly increased in size (Fig. 12–16A). Cerebellar, brainstem, and cerebral astrocytomas may additionally be seen. High T2 signal intensity foci may be identified within the peduncles or deep gray matter of the cerebellum, brainstem, basal ganglia (particularly, globus pallidus), and supratentorial white matter (Fig. 12–16B). The nature of these lesions remains unresolved.

Fig. 12–16.

Noncontrast T1-weighted A and T2-weighted B images in a 10-year-old male with neurofibromatosis. A Bulbous enlargement of the optic chiasm is present (arrow), suggesting an optic glioma. B Foci of increased T2 signal abnormality are demonstrated within the globus palladi (arrows).

EXERCISE 12-2: STROKE

Clinical Histories:

Case 12-3. A 47-year-old female presents with diabetes mellitus and recent development of left-sided hemiplegia. Axial images from an uninfused head CT examination are shown in Fig. 12–17A,B .

Case 12-4. A 66-year-old woman presents with gradual onset of nausea, dizziness, and ataxia. The patient became comatose 24 hours after the onset of symptoms. Axial T2-weighted and sagittal T1-weighted images are shown in Fig. 12–18A,B .

Case 12-5. A 42-year-old female hypertensive renal transplant patient presents with acute mental status changes and left hemiparesis. A single axial image from a noncontrast head CT scan is shown in Fig. 12–19.

Fig. 12–17.

Fig. 12–18.

Fig. 12–19.

Questions:

12-3. In Case 12-3, what is the most likely diagnosis?

A. Intracranial abscess

B. Arachnoid cyst

C. Metastatic brain tumor

D. Primary brain tumor

E. Cerebral infarction

12-4. In Case 12-4, what is the likely cause of the patient's problem?

A. Brain stem infarction

B. Brain stem compression from cerebellar infarction

C. Brain stem tumor

D. Cerebellar astrocytoma

E. Posterior fossa hemorrhage

12-5. In Case 12-5, what is the most likely diagnosis?

A. Thalamic glioma

B. Subarachnoid hemorrhage

C. Metastatic disease

D. Hypertensive hemorrhage in the basal ganglia

E. Cerebral contusion

Radiologic Findings:

12-3. In this case, the axial CT image (Fig. 12–17A ) demonstrates a well-defined area of hypodensity (arrows) in the right middle cerebral artery (MCA) territory. There is associated mass effect on the surrounding brain parenchyma with a corresponding shift of the midline structures to the left. In a more inferior axial image (Fig. 12–17B ), note the bright right MCA (arrow) corresponding to an acute thrombus in the main trunk of this vessel. (E is the correct answer to Question 12-3.)

12-4. In this case, the axial T2-weighted MR image (Fig. 12–18A ) shows areas of increased T2 signal (arrows) corresponding to edema within the cerebellum. A sagittal T1-weighted image (Fig. 12–18B ) shows a swollen cerebellum, as well as upward transtentorial (arrowhead) and downward tonsillar (curved arrow) herniation of cerebellar tissue. Also note compression of the brain stem (small arrows) and fourth ventricle (asterisk). These changes are compatible with a recent cerebellar infarction with brain stem compression caused by the swollen cerebellum. (B is the correct answer to Question 12-4.)

12-5. In this case, an axial CT scan (Fig. 12–19) demonstrates a right basal ganglia hematoma (large black arrow) with intraventricular extension (small black arrows). Note the shift of midline structures (white arrows). This is most likely secondary to the patient's known hypertension. (D is the correct answer to Question 12-5.)

Discussion:

Stroke is a lay term for acute neurologic disfunction. The usual image of a stroke patient is that of an elderly individual with a hemiparesis, often associated with abnormal speech. There are actually many different causes of stroke. These include cerebral infarction, intracerebral hemorrhage, subarachnoid hemorrhage, and miscellaneous causes such as dural sinus occlusion with associated venous infarction. Although these conditions may have similar clinical presentations, they have different treatments and prognoses.

The vast majority of strokes are cerebral infarctions associated with atherosclerosis. The radiologic manifestations of cerebral infarction vary with time. The head CT scan of the patient in Case 12-3 was obtained several days after the onset of symptoms and shows typical findings of a subacute infarct in a major vascular territory, in this case the right middle cerebral artery region. By this time, the infarct is a very well-defined area of low attenuation compared to normal surrounding brain. There is associated mass effect from the edematous tissue. Acute infarcts (less than 24 hours since onset of symptoms) may be invisible on head CT scans, although diffusion-weighted MR imaging often demonstrates brain abnormalities within several hours (or less) of symptom onset. Subtle changes on head CT scans in acute infarction can sometimes be seen, but may be overlooked if the examination is not closely scrutinized. Sometimes the only apparent change on CT scans is a subtle loss of gray matter white matter differentiation in the area of infarction. CT scanning is performed in acute cerebral infarction because scans can be quickly obtained, and CT is a very good test for identifying intracranial hemorrhage, an important finding for management considerations. If the infarct is not obvious on the initial CT scan, an MR scan is usually obtained to verify high clinical suspicion.

An acute or subacute infarction will exhibit a diffusion signal abnormality that reflects the restricted movement of water molecules, and it typically persists for 1 to 2 weeks within infarcted tissue. Leptomeningeal enhancement that extends into the cortical sulci may be seen within several days of a cerebral infarction (Fig. 12–20). Parenchymal enhancement is commonly identified within the gray matter, which usually has a band-like, tubular, or gyriform appearance. Solid or ring-enhancing areas, as well as more amorphous-appearing patterns of enhancement, can occasionally occur.

Fig. 12–20.

A Noncontrast axial CT image of a subacute left MCA infarction in a middle-aged man with right-sided hemiparesis. A well-defined area of decreased attenuation is seen within the left MCA distribution (arrows). B The subsequent diffusion-weighted MR image demonstrates a well-circumscribed, wedge-shaped area of increased signal (arrows) corresponding to this CT signal abnormality. C Postcontrast coronal T1-weighted MR image reveals linear enhancement along several cortical sulci (closed arrows), with associated band-like enhancement involving the adjacent cortex (open arrow).

Case 12-4 illustrates an important point to consider when deciding which test to order in the setting of acute stroke. In this case, the patient's symptoms were worrisome for a brain stem process. CT scanning of the brain stem and posterior fossa is frequently degraded by streak artifacts emanating from the dense bone of the skull base. Subtle (and sometimes not so subtle) abnormalities may not be apparent. Therefore, for most neurologic conditions that involve the brain stem or posterior fossa, MR scans are much better at depicting an abnormality. Notice that the patient in Case 12-4 did not in fact have a brain stem infarct, as was suspected clinically, but rather had brain stem compression from a large cerebellar infarct.

Case 12-5 illustrates how essential an imaging examination is in managing stroke. The patient had signs and symptoms of an acute cerebral infarction. The CT scan demonstrated an obvious basal ganglia hemorrhage, probably secondary to the patient's hypertension. Management of these two conditions is considerably different. Hypertension is the main cause of nontraumatic intracranial hemorrhage. In adults, these hemorrhages typically occur in the putamen/external capsule. Other locations for hypertensive hemorrhage include the thalamus, pons, cerebellum, and, rarely, subcortical white matter. Acute parenchymal hematomas, as in this case, are usually hyperdense on CT scans. With time these lesions become darker and eventually appear as round or slit-like cavities. The MR imaging appearance of a parenchymal hematoma is complex and depends largely on the presence of hemoglobin breakdown products within the clot.

EXERCISE 12-3: BRAIN TUMORS

Clinical Histories:

Case 12-6. A 33-year-old Hispanic man presents with a syncopal episode and involuntary tremors. Noncontrast sagittal T1-weighted and axial T2-weighted images, as well as postcontrast axial T1-weighted images are shown in Fig. 12–21A–C.

Case 12-7. A 48-year-old woman presents with a history of headaches and seizures. Initial coronal T2-weighted FLAIR and axial contrast-enhanced T1-weighted images were obtained (Fig. 12–22A,B ).

Case 12-8. A 58-year-old man presents with a history of lung cancer and mental status changes. A contrast-enhanced axial CT scan and a gadolinium-enhanced axial T1-weighted MR image are shown in Fig. 12–23A,B .

Fig. 12–21.

Fig. 12–22.

Fig. 12–23.

Questions:

12-6. In Case 12-6, what is the most likely diagnosis?

A. Extra-axial brain tumor

B. Intra-axial brain tumor

C. Frontal contusion

D. Subdural hematoma

E. Encephalocele

12-7. In Case 12-7, what is the most likely cause of the patient's symptoms?

A. Multiple sclerosis

B. Inner ear abnormality

C. Intraventricular meningioma

D. Hematoma

E. Malignant brain tumor

12-8. In Case 12-8, what is the most likely explanation for the patient's mental status changes?

A. Metastatic disease

B. Intracranial hemorrhage

C. Small infarcts

D. Sarcoidosis

E. Arteriovenous malformation

Radiologic Findings:

12-6. In this case, the sagittal T1-weighted image before contrast administration (Fig. 12–21A ) shows anextra-axial, left frontal convexity mass (arrows). This homogeneous-appearing, smoothly marginated mass is isointense to the normal gray matter, and is sometimes difficult to differentiate from normal brain tissue on unenhanced T1 images. On T2-weighted imaging (Fig. 12–21B ), the mass has a heterogeneous appearance, but is predominantly isointense to gray matter. The mass is circumscribed by a thin rim (pseudocapsule) of increased T2 signal (long arrows), as well as marginated by a more peripherally located band of T2 signal hyperintensity along its medial and posterior borders (short arrows). There is distortion of the adjacent brain parenchyma, with compression of the left lateral ventricle, and a mild shift of the midline structures to the right. Following intravenous Gd-DTPA administration (Fig. 12–21C ), the mass enhances uniformly (arrows), and dural tails are observed (arrowheads), allowing easy identification. These features are fairly typical of a meningioma. (A is the correct answer to Question 12-6.)

12-7. In this case, a coronal T2-weighted FLAIR MR image (Fig. 12–22A ) demonstrates a large area of T2 signal hyperintensity involving the inferior frontal regions (large white arrows) and right temporal lobe (small white arrow), with extension into the corpus callosum (curved arrows). On the infused axial view, at the level of the body of the corpus callosum (Fig. 12–22B ), subtle, ill-defined enhancement is present within the right cerebral hemisphere (arrowhead) with patchy enhancement (arrows) extending into the body of the corpus callosum. This is one appearance of a malignant brain tumor, in this case, an anaplastic oligodendroglioma. (E is the correct answer to Question 12-7.)

12-8. In this case, a contrast-enhanced axial CT scan (Fig. 12–23A ) shows no definite abnormality. A gadolinium-enhanced axial T1-weighted MR image (Fig. 12–23B ) shows multiple enhancing lesions (arrows) within the brain parenchyma. In a patient with known lung cancer, metastatic disease is the most likely explanation for multiple intracranial enhancing lesions (A is the correct answer to Question 12-8.)

Discussion:

Brain tumors can be classified in a variety of ways. The traditional classification of intracranial neoplasms is based on histology. In this system, brain tumors are either primary (they arise from the brain and its linings) or secondary (they arise from somewhere outside the CNS, i.e., metastases). Primary tumors, which account for approximately two-thirds of all brain neoplasms, can be subdivided into glial and nonglial tumors. Secondary tumors, especially from lung and breast cancer, account for the remaining one-third of brain neoplasms. Metastases are most commonly parenchymal, but can also involve the skull and meninges.

Brain tumors can also be classified according to patient age and general tumor location (i.e., adult or child, supratentorial or infratentorial). Finally, brain tumors can be classified according to the specific anatomic region involved. For example, we can generate lists of brain tumors that specifically affect the pineal or the pituitary regions.

Case 12-6 illustrates a useful principle for interpreting studies of patients with suspected brain tumors. It is very important to first decide whether a mass is within the brain parenchyma (intra-axial) or outside the brain (extra-axial). Extra-axial masses usually turn out to be meningiomas, many of which can be removed surgically with a very low incidence of recurrence. Intra-axial masses frequently turn out to be astrocytomas, and the prognosis is less favorable.

The patient in Case 12-6 has an extra-axial, dural-based, frontal convexity mass that markedly enhances with Gd-DTPA. Meningiomas comprise 15% to 20% of intracranial tumors, occur predominantly in females, and exhibit a peak age incidence of 45 years. They are the most common nonglial primary CNS tumors. They can occur anywhere within the head but typically occur along the dural venous sinuses. The parasagittal region and cerebral convexities are the most common locations. Anterior basal or olfactory groove meningiomas account for 5% to 10% of intracranial meningiomas. Anosmia results from involvement of the olfactory tracts by the tumor. These expansile lesions are slow growing, and the ensuing mass effect on the adjacent brain parenchyma is gradual. The absence of reactive edema in a subset of these lesions can be seen as a result of their slow growth. These masses usually demonstrate intense and uniform enhancement, independent of tumor size. A layer of thickened dural enhancement ("dural tail") is commonly seen extending away from the base of the meningioma. In many cases, this finding represents reactive thickening without tumor involvement.

Case 12-7 demonstrates a large, infiltrating (aggressive or high-grade) glioma involving the majority of the right frontotemporal lobe, with extension into the corpus callosum. While there is some overlap of the MR imaging features characteristically seen with these invasive neoplasms and their less aggressive (lower grade) counterparts, the imaging features of higher grade neoplasms, on the whole, are distinctly different from those seen with lower grade lesions. High-grade gliomas, namely, anaplastic astrocytomas and oligodendrogliomas (as in this case), as well as glioblastoma multiforme (the most highly malignant glioma), demonstrate heterogeneous signal characteristics, generally a reflection of the variable cellularity, in addition to the presence of necrosis, hemorrhage, and cystic foci. Calcification and hemorrhage are more common in oligodendrogliomas, often accompanied by cyst formation and necrosis. The spectroscopic findings of decreased NAA and increased choline suggest decreased neuronal/axonal density and increased breakdown of cell membranes (Fig. 12–24A,B ).

Fig. 12–24.

The same patient as in Figure 12-22. A At a more inferior level, the patchy, heterogeneous enhancement of this mass within the right inferior frontal/temporal regions is better appreciated. A region-of-interest or volume element (i.e., voxel) was centered within the enhancing tumor volume, and an MR spectrum was obtained. B MR spectrum. The NAA peak is abnormally decreased (short arrow at 2.0), and the choline signal is elevated (long arrow at 3.2), supporting the diagnosis of a malignant brain tumor.

Oligodendrogliomas account for about 5% of primary gliomas, occurring most frequently within the frontal lobe and often involving the cortex. The majority of patients present with seizures. On the other hand, glioblastoma multiforme is the most common primary malignant brain neoplasm and occurs most frequently in patients older than 50 years of age. Patients with glioblastoma multiforme present with neurologic deficits or new-onset seizures. The prognosis in these latter cases is dismal; postoperative survival averages 8 months.

On T2-weighted scans, these high-grade masses usually exhibit heterogeneous signal characteristics, with areas of high T2 signal attributable to tumor tissue, necrosis, cysts, and reactive edema, while regions of low signal may reflect hemorrhage or calcification. The corresponding tissue pathology of this region often shows tumor cells residing within and extending beyond the surrounding edema. Enhancement is highly variable within anaplastic oligodendrogliomas. Other types of malignant gliomas, such as glioblastoma multiforme, typically demonstrate intense enhancement. The corpus callosum is often involved by a high-grade glial tumor, which may grow medially from an adjacent hemispheric source or may arise independently within this structure. "Wings" may extend symmetrically or asymmetrically into both cerebral hemispheres, exhibiting a butterfly-type appearance (Fig. 12–25), appropriately termed butterfly glioma.

Fig. 12–25.

Coronal contrast-enhanced T1-weighted MR image of a glioblastoma multiforme in a 76-year-old woman with a 6-month history of progressive gait ataxia and frequent falling is shown. An enhancing mass (white arrows) extends through the corpus callosum (black arrows) into both hemispheres.

Case 12-8 illustrates a very important point to remember when working up patients with suspected metastatic disease to the brain: MR imaging is considerably more sensitive than CT in detecting metastases. This is not a trivial point, since surgical resection of single, not multiple, brain lesions is sometimes performed. Conversely, the successful application of radiotherapy protocols relies on sensitively and accurately detecting the entire metastatic tumor burden. Metastatic disease to the brain has a variety of manifestations, the most common being parenchymal involvement. Typical hematogenous brain metastases demonstrate solid or ring-like enhancement on CT or MR scans, occur near gray matter white matter junctions, and are usually surrounded by a marked amount of edema. They most commonly metastasize from lung or breast primaries.

EXERCISE 12-4: INTRACRANIAL INFECTIONS

Clinical Histories:

Case 12-9. A 75-year-old man presents with a history of recurrent lymphoma complicated by multiple infections and new mental status changes. Postcontrast axial T1-weighted and diffusion-weighted MR images are shown in Fig. 12–26A,B .

Case 12-10. A 4-year-old girl presents with lethargy and seizure activity. An axial T2-weighted MR image of the brain is shown in Fig. 12–27.

Case 12-11. A 43-year-old man presents with headache and weakness. An axial contrast-enhanced T1-weighted MR image is shown in Fig. 12–28.

Fig. 12–26.

Fig. 12–27.

Fig. 12–28.

Questions:

12-9. In Case 12-9, what is the most likely diagnosis?

A. Frontal contusion

B. Aneurysm with intraventricular hemorrhage

C. Parietal lobe abscess

D. Intracranial lymphoma

E. Cerebritis

12-10. In Case 12-10, the location of the abnormality is pathognomonic for which type of infection?

A. Toxoplasmosis

B. Tuberculosis

C. Cryptococcus

D. Herpes

E. Staphylococcus

12-11. In Case 12-11, the major differential diagnosis for this lesion is toxoplasmosis versus

A. cryptococcus.

B. intracranial lymphoma.

C. sarcoidosis.

D. metastatic disease.

E. cytomegalovirus (CMV).

Radiologic Findings:

12-9. In this case, the contrast-enhanced MR scan (Fig. 12–26A ) shows a ring-enhancing lesion (arrows) in the left parietal lobe with decreased surrounding T1 signal. A diffusion signal abnormality is present on the corresponding diffusion-weighted image (Fig. 12–26B ) within the central aspect of the lesion, found to be compatible with an area of restricted water motion. The patient's history is compatible with an intracranial infection, and the demonstrated MR imaging findings favor an abscess. (C is the correct answer to Question 12-9.)

12-10. In this case, the T2-weighted MR image (Fig. 12–27) shows high-signal abnormality in the medial aspect of the right temporal lobe (arrows) and within the inferior right frontal lobe (curved arrow). These changes are commonly seen in patients with herpes encephalitis. (D is the correct answer to Question 12-10.)

12-11. In this case, multiple enhancing lesions are present within the basal ganglia, especially on the right (arrows), on the gadolinium-enhanced T1-weighted MR image (Fig. 12–28). The most common lesions with this appearance in an HIV-positive patient are toxoplasmosis and intracranial lymphoma. (B is the correct answer to Question 12-11.) The patient markedly improved after antitoxoplasmosis therapy, and the lesions shown on the MR image disappeared.

Discussion:

A host of infectious diseases can involve the brain and its coverings. Because the CNS has a limited number of ways of responding to an infectious agent, many intracranial infections appear identical on neuroimaging studies. It is, therefore, very important to closely correlate the imaging findings with the clinical presentation and other diagnostic tests, such as lumbar puncture or stereotactic brain aspiration.

For our purposes, it is useful to classify CNS infections according to the intracranial compartment involved, especially since this has treatment implications. Intracranial infections can be either parenchymal or extraparenchymal. Parenchymal manifestations include cerebritis/abscess and encephalitis. Extraparenchymal disease includes epidural abscess, subdural empyema, and leptomeningitis. Bacterial, viral, fungal, and parasitic agents can all affect the CNS. Although a few infectious agents preferentially involve a particular anatomic compartment of the CNS, most are not site specific.

Case 12-9 demonstrates the classic ring-enhancing lesion of an abscess, in this case, due to nocardia. No specific features of this abscess distinguish it from a typical pyogenic abscess. The diffusion signal abnormality has been postulated to arise from restricted water motion in the presence of viscous, prurulent material within the abscess cavity, and it can mimic an area of acute ischemia. Cerebral infection by nocardia usually arises from a pulmonary focus in an immunocompromised host. Similarly, most pyogenic abscesses are the result of hematogenous dissemination from a non-CNS source. Pyogenic brain abscesses can also result from direct extension of an infectious process from an adjacent area (e.g., sinusitis or mastoiditis) or from trauma (e.g., penetrating wound or surgery).

Abscesses usually occur at gray matter white matter junctions, although they can occur anywhere in the brain. Patients frequently present with seizures or symptoms related to intracranial mass effect. If abscesses develop near the brain surface, they may rupture into the subarachnoid space, producing a meningitis; they may also produce a ventriculitis if they rupture into the ventricular system. Most abscesses are treated surgically.

Herpes encephalitis (Case 12-10) is caused by the herpes simplex virus (HSV). Older children and adults are usually infected by HSV-1, either primarily or as a result of reactivation of a latent virus. The ensuing necrotizing encephalitis in this condition typically involves the temporal and inferior frontal lobes, insular cortex, and cingulate gyrus. Focal abnormalities of attenuation (on CT) or signal (on MR) in these characteristic locations, often with enhancement after contrast administration, are practically pathognomonic of HSV-1 encephalitis. Early diagnosis of this condition is extremely important, because antiviral therapy can significantly affect patient outcome.

Neonatal herpes simplex infection differs from infection in the older child and adult. The offending organism is usually HSV-2, which may be acquired in utero or during birth from mothers with genital herpes. HSV-2 infection can produce severe destructive changes within the developing brain. Unlike HSV-1 infection in older children and adults, neonatal herpes encephalitis can involve any area of the brain, having no predilection for the temporal lobe.

Patients with AIDS (Case 12-11) commonly develop intracranial infections during the course of their disease. HIV itself can directly infect the CNS, producing encephalopathy in up to 60% of AIDS patients. The most common neuroimaging finding in HIV encephalopathy is cerebral atrophy, often with patchy white matter hypointensity (on CT) or increased T2 signal (on MR imaging) from demyelination and gliosis (Fig. 12–29). Other common CNS infections in the immunocompromised AIDS patient include toxoplasmosis, cryptococcosis, and progressive multifocal leukoencephalopathy (from a papovavirus infection).

Fig. 12–29.

An 8-year-old girl with AIDS and new onset of seizures. Axial T2-weighted image shows white matter high signal (arrows). Also note the diffuse prominence of gyri and sulci (arrowheads) and sylvian fissures (asterisks), compatible with cerebral atrophy.

Toxoplasmosis usually presents as multiple lesions of varying size, and demonstrates ring enhancement with surrounding edema on CT or MR imaging (Fig. 12–28). Lesions commonly occur in the basal ganglia or at the gray matter white matter junction within the cerebral hemispheres. Individual masses may have a solid appearance or demonstrate central necrosis or hemorrhage. The enhancement pattern is variable; both rim-enhancing and more solidly enhancing lesions can be seen. Their appearance is almost identical to that of primary intracranial lymphoma, another common intracranial condition in AIDS. Metabolic studies, such as PET or SPECT scans (no increase in 18F-FDG activity with toxoplasmosis, increased with lymphoma), MR spectroscopy (no choline elevation in toxoplasmosis, elevated in lymphoma), and perfusion-weighted sequences (lower cerebral blood volume in toxoplasmosis) may assist in distinguishing these pathologies.

Meningitis is the most frequent manifestation of cryptococcosis in AIDS, although parenchymal lesions, termedcryptococcomas, are occasionally encountered. In progressive multifocal leukoencephalopathy, extensive areas of white matter demyelination are shown on MR imaging. A number of other intracranial infections can occur in AIDS patients and the reader is referred to the Bibliography at the end of this chapter for sources of further information.

EXERCISE 12-5: HEAD TRAUMA

Clinical Histories:

Case 12-12. A young man who has been in a motor vehicle accident presents with a head injury. Soft-tissue window from an axial non-contrast head CT scan is shown in Fig. 12–30.

Case 12-13. A 24-year-old man presents with multiple facial fractures and frontal scalp soft-tissue swelling resulting from a motor vehicle accident. Axial noncontrast head CT images are shown in Fig. 12–31.

Fig. 12–30.

Fig. 12–31.

Questions:

12-12. In Case 12-12, what is the diagnosis?

A. Subdural hematoma

B. Cerebral contusion

C. Epidural hematoma

D. Meningioma

E. Subdural hygroma

12-13. In Case 12-13, what is the main radiologic finding?

A. Subdural hematoma

B. Epidural hematoma

C. Duret hemorrhage

D. Cerebral contusions

E. Shearing injuries

Radiologic Findings:

12-12. In this case (Fig. 12–30), a predominantly high-density, extra-axial, hemorrhagic collection (arrows) is producing mass effect on the left temporoparietal lobe on an unenhanced head CT scan. The biconvex appearance of this lesion is typical of an epidural hematoma, which is an acute finding in this case. (C is the correct answer to Question 12-12.)

12-13. In this case, multiple areas of increased attenuation within the frontal lobes, especially on the left (arrows), are seen in Fig. 12–31. These areas correspond to multiple hemorrhagic contusions involving the brain parenchyma. (D is the correct answer to Question 12-13.)

Discussion:

Intracranial abnormalities in head trauma can be classified as either primary or secondary. Primary lesions occur at the moment of injury and include skull fractures, extra-cerebral hemorrhage (e.g., epidural or subdural hematomas, subarachnoid hemorrhage), and intracerebral hemorrhage (e.g., brain contusion, brain stem injury, diffuse axonal injury).

The secondary effects of head trauma are actually complications of the primary intracranial injury. Elevated intracranial pressure and cerebral herniations are responsible for most of the secondary effects of head trauma, which in many cases may be more devastating to the patients than the initial injury.

Epidural hematoma is usually associated with skull fractures that lacerate the middle meningeal artery or a dural sinus. Up to one-half of patients with epidural hematomas have a lucid interval after the head trauma occurs. On CT, epidural hematomas usually appear as biconvex, high-attenuation, extra-axial masses. Most are located in the temporoparietal area. Underlying skull fractures are common. Intracranial brain herniation may also be a prominent feature in this condition. One important imaging feature in epidural hematomas is that they do not cross skull sutures.

Subdural hematoma, on the other hand, is usually a crescent-shaped extra-axial collection that may cross suture lines (Fig. 12–32). These lesions are more lethal than are epidural hematomas; the subdural hematoma mortality rate is greater than 50%. CT can usually, but not always, distinguish between epidural hematomas and subdural hematomas. Subdural hematomas are a commonly identified abnormality in the abused child (nonaccidental trauma). CT scans are obtained to detect the presence of subdural hematomas (Fig. 12–33). Brain MR imaging, however, can more sensitively delineate small extra-axial hematomas, subdural hematomas of varying ages, and coexisting cortical contusions or shearing injuries. A shearing injury (or diffuse axonal injury) is associated with an overall poor prognosis, and it is recognized as small petechial hemorrhages at the gray white junction and in the corpus callosum. Interhemispheric (para- and intrafalcial) subdural hematomas may arise from tearing of bridging veins along the falx cerebri in shaking injuries and is nearly pathognomic for nonaccidental trauma (Fig. 12–33B ). Retinal hemorrhages may be present and are also suspicious, especially if bilateral. In addition, cerebral ischemia/infarction and multiple, complex, unexplained skull fractures may be associated findings.

Fig. 12–32.

Uninfused axial CT image in a middle-aged patient following a fall. A large, crescentic, extra-axial hemorrhagic collection layers over the right lateral convexity (arrows), consistent with an acute subdural hematoma. There is associated mass effect on the adjacent brain parenchyma, with effacement of the cortical gyri, compression of the right lateral ventricle, and shift of the midline structures to the left.

Fig. 12–33.

Noncontrast axial CT images A, B in a 21-day-old male following nonaccidental trauma. Large, bilateral subdural hematomas layer over the tentorium cerebelli in A (closed arrows) and within the interhemispheric fissure in B(arrow). In addition, a small amount of subarachnoid hemorrhage is seen within the quadrigeminal plate cistern in A(open arrows), as well as within the left lateral ventricle (not shown). Loss of the normal cerebral gray white differentiation is demonstrated. These features are pathognomonic for nonaccidental trauma with diffuse anoxic insult.

Cerebral contusions (Case 12-13) are the second most common form of brain parenchymal injury in primary head trauma. (Diffuse axonal injury is the most common parenchymal injury.) Cerebral contusions can be thought of as brain bruises. They result either from the brain striking a bony ridge inside the skull during rapid acceleration/deceleration, as occurs in a motor vehicle accident, or from a depressed skull fracture. These lesions tend to occur in particular anatomic locations, especially the undersurfaces and poles of the frontal and temporal lobes. CT scans show areas of low attenuation (edema) and hemorrhage at the site of injury. Delayed hemorrhage, 1 to 2 days after a head injury, is common with contusions.

EXERCISE 12-6: INTRACRANIAL VASCULAR ABNORMALITIES

Clinical Histories:

Case 12-14. A 59-year-old woman presents with a severe headache. An axial head CT image and a cerebral arteriogram are shown in Fig. 12–34A,B ).

Case 12-15. An 11-year-old boy presents with an acute decline in mental status. Axial head CT image and cerebral arteriogram are shown in Fig. 12–35A,B ).

Fig. 12–34.

Fig. 12–35.

Questions:

12-14. In Case 12-14, what is the reason for the abnormality on the CT scan?

A. Cerebral aneurysm

B. Arteriovenous malformation

C. Head trauma

D. Carotid dissection

E. Vasculitis

12-15. In Case 12-15, what is the reason for the abnormality on the CT scan?

A. Cerebral aneurysm

B. Arteriovenous malformation

C. Head trauma

D. Carotid dissection

E. Vasculitis

Radiologic Findings:

12-14. In this case, the CT scan (Fig. 12–34A ) shows extensive subarachnoid hemorrhage filling the basal cisterns (arrows). Curvilinear calcifications (arrowhead) are present in the region of the anterior communicating artery. Oblique frontal view from a carotid arteriogram (Fig. 12–34B ) demonstrates the source of this bleeding: a large aneurysm (large arrow) between the anterior cerebral arteries (small arrows). Also note the internal carotid (arrowhead) and middle cerebral (curved arrow) arteries. (A is the correct answer to Question 12-14.)

12-15. In this case, the CT scan (Fig. 12–35A ), shows intraparenchymal (large arrow) and intraventricular hemorrhage (small arrows). The lateral view of a carotid arteriogram (Fig. 12–35B ) demonstrates a tangle of blood vessels typical of an arteriovenous malformation (large arrow) with early draining veins (small arrows). Note the internal carotid artery (curved arrow). (B is the correct answer to Question 12-15.)

Discussion:

Cerebrovascular disorders (strokes) were discussed in Exercise 12-2, which dealt mainly with cerebral infarction secondary to atherosclerosis. For information on other causes of cerebral infarction, the reader is referred to the bibliography at the end of this chapter. This section addresses two other common vascular conditions affecting the CNS: aneurysms and vascular malformations.

Most cerebral aneurysms, like Case 12-14, are saccular or "berry" aneurysms. These focal arterial dilatations tend to occur at cerebral arterial branch points. They have traditionally been thought to develop at congenitally weak areas of a blood vessel wall. Recent evidence, however, has questioned this view, and many now believe that saccular aneurysms are probably acquired lesions from abnormal hemodynamic stresses that damage the arterial wall.

Intracranial aneurysms are usually asymptomatic until they rupture, at which time the patient typically presents with a severe headache resulting from subarachnoid hemorrhage (SAH). The vast majority of nontraumatic SAHs occur as a result of aneurysm rupture. CT is very good at demonstrating SAH. Patients usually undergo cerebral arteriography whenever nontraumatic SAH is detected.

Common locations for intracranial aneurysms include the anterior communicating artery, the internal carotid artery at the origin of the posterior communicating artery, and the middle cerebral artery trifurcation. Posterior fossa aneurysms are less common; they make up only around 10% of all intracranial aneurysms.

Vascular malformations can be divided into four major types: true arteriovenous malformations (as demonstrated in Case 12-15), cavernous hemangiomas, venous angiomas, and capillary telangiectasias. Arteriovenous malformations (AVMs) are congenital lesions consisting of a tangle of abnormal blood vessels, usually within the brain parenchyma, that are fed by enlarged cerebral arteries and drained by dilated, tortuous veins. Because there is no normal intervening brain parenchyma for the blood to flow through, blood is rapidly shunted from the arterial to the venous side. This shunting is dramatically demonstrated on cerebral arteriography. Patients with AVMs usually present with intracranial hemorrhage or seizures. MR imaging or contrast-enhanced CT can demonstrate the tortuous vascular channels of most AVMs, although cerebral arteriography is the definitive study in this condition.

The other intracranial vascular malformations have very characteristic appearances on MR imaging, although they are frequently invisible on cerebral arteriography. Patients with these "low-pressure" malformations can present with headaches, seizures, or, rarely, intracranial hemorrhage. Many of these lesions, however, are incidentally discovered on MR scans performed for other reasons.

EXERCISE 12-7: WHITE MATTER DISEASES

Clinical Histories:

Case 12-16. A 23-year-old woman presents with numbness, weakness, and blurred vision. Axial T2-weighted and coronal FLAIR MR images of the brain are shown in Fig. 12–36A,B .

Case 12-17. A 77-year-old woman presents with a long history of hypertension and recent onset of dementia. An axial T2-weighted MR image is shown in Fig. 12–37.

Fig. 12–36.

Fig. 12–37.

Questions:

12-16. In Case 12-16, what is the most likely diagnosis?

A. Pseudotumor cerebri

B. Metastatic disease

C. Septic emboli

D. Radiation necrosis

E. Multiple sclerosis

12-17. In Case 12-17, what is most likely responsible for the abnormalities seen on the MR image?

A. Cardiac arrhythmia

B. Chronic hypertension

C. Remote trauma

D. Hepatic failure

E. Carbon monoxide poisoning

Radiologic Findings:

12-16. In this case, the axial T2-weighted and coronal FLAIR MR images (Fig. 12–36A,B ) show multiple foci of increased T2 signal within the white matter (arrows). These lesions are quite characteristic of multiple sclerosis. (E is the correct answer to Question 12-16.) The patient's visual difficulties were due to optic neuritis, a common abnormality in multiple sclerosis.

12-17. In this case (Fig. 12–37), there are patchy areas of increased T2 signal (arrows) within the periventricular white matter. Usually seen in elderly hypertensive patients, these lesions correspond to focal areas of demyelination secondary to deep white matter ischemia. (B is the correct answer to Question 12-17.)

Discussion:

Diseases that primarily affect the cerebral white matter have a host of causes. Unfortunately, very few of these conditions have specific appearances on CT or MR scans. Neuroimaging is usually performed to determine whether there are changes within the brain that are compatible with one of the white matter diseases and to rule out other conditions that might mimic white matter disease.

White matter diseases include both inherited and acquired conditions. They can be further subdivided into demyelinating conditions (destruction or injury of normally formed myelin) and dysmyelinating conditions (abnormal formation or maintenance of myelin, usually because of an enzyme deficiency). The dysmyelinating conditions are rare and, for the most part, include the leukodystrophies, such as adrenoleukodystrophy and metachromatic leukodystrophy. Although the MR appearance can be striking in some of these diseases, it is often nonspecific. These conditions are not discussed here.

Multiple sclerosis (MS) (Case 12-16) is the most common demyelinating disease. Because there is no generally accepted etiology for MS, it is also referred to as a primary demyelinating disease. Secondary demyelinating conditions are those caused by a known agent or event. MS usually occurs in young adults and more often in women than men (approximately 2:1). The disease is characterized by a relapsing and remitting course and by varying neurologic symptoms, depending on the location of the lesion within the CNS. Although diagnosis of MS is usually based on clinical criteria, MR imaging can be a very helpful confirmatory test. Typical MS plaques appear as ovoid, T2 signal hyperintensities within the periventricular and deep white matter. Lesions are also common within the corpus callosum, brain stem, cerebellar peduncles, spinal cord, and optic nerves. MS plaque enhancement on gadolinium-infused MR images suggests active disease (i.e., breakdown of the BBB). Confluent areas of T2 signal abnormality in the periventricular white matter are common in severe cases.

Ischemic demyelination (Case 12-17) is usually seen in patients with small-vessel disease (such as from long-standing hypertension). This condition, also called leukaraiosis (white matter softening), occurs because of hypertension-induced arteriolar sclerosis of penetrating medullary arteries that supply the deep white matter of the brain. This leads to a reduction in white matter blood flow with accompanying ischemic demyelination. This condition occurs most commonly in older patients and is associated with small-vessel brain infarcts (lacunar infarcts). MR imaging usually demonstrates patchy areas of increased T2 signal in the deep white matter. The lesions are often bilaterally symmetric and periventricular in distribution.

BIBLIOGRAPHY

Aksoy FG et al. Dynamic contrast-enhanced brain perfusion imaging: technique and clinical applications. Semin Ultrasound CT MRI. 2000;21:462–467. [PMID: 11138635]

Atlas SW, ed. Magnetic Resonance Imaging of the Brain and Spine. New York: Raven Press; 2002.

Cullen SP et al. Dynamic contrast-enhanced computed tomography of acute ischemic stroke: CTA and CT perfusion.Semin Roentgenol. 2002;37:192–205. [PMID: 12226898]

Grossman RI, Yousem DM. Neuroradiology: The Requisites. St. Louis; Mosby; 2003.

Liu H et al. MR-guided and MR-monitored neurosurgical procedures at 1.5T. JCAT. 2000;24:909–918. [PMID: 11105712]

Philips CD et al. CTA and MRA in the evaluation of extracranial carotid vascular disease. Radiol Clin N Am. 2002;40:783–798.

Orrison WW et al. Functional Brain Imaging. St. Louis: Mosby; 1995.

Osborn AG. Diagnostic Neuroradiology. St. Louis: Mosby; 1994.

Yock DH. Magnetic Resonance Imaging of CNS Disease. St. Louis: Mosby; 2002.



If you find an error or have any questions, please email us at admin@doctorlib.org. Thank you!